× Binary Unit Round-off Roundoff Error Number System with base \(\beta\) Floating point error analysis Orders of Convergence Newton's Method Error Analysis for Newton's Method Theorems on Newton's Method Horner's Algorithm Contractive Mapping Polynomial Interpretation Hermite Interpolation Change of Intervals Gaussian Quadrature Error Analysis for Gaussian Quadrature Composite Formulae \(\theta\)-Method Error Initial Value Problem Stability Local Truncation Error for Multistep Methods Schur's Form Hermite Matrix QR Factorization Householder LU Decomposition Norm Positive Definite Gerschgorin Theorem Rayleigh Quotient August 2017 August 2018 January 2019 August 2019 January 2020 August 2020 January 2021 August 2021 January 2022 August 2022 January 2023 August 2023 January 2024 References
☰ Menu

My name is Hanzhang Yin, and I have developed this website as a resource to facilitate the review of key concepts in abstract algebra, and it is not fully completed. Feel free to email me at hanyin@ku.edu if there are any errors or suggestions for improvement.

2023 (January) Numerical Analysis Qualifying Examination

Problem 4. For \( A \in \mathbb{R}^{n \times n} \) and \( A = A^T \), Rayleigh Quotient Iteration is defined by:

select \( \vec{v}^{(0)} \in \mathbb{R}^n \) such that \( \|\vec{v}^{(0)}\|_2 = 1 \)

for \( k = 1, 2, \ldots \)

compute \( \lambda^{(k-1)} = [\vec{v}^{(k-1)}]^T A \vec{v}^{(k-1)} \)

solve \( (A - \lambda^{(k-1)}I)\vec{w} = \vec{v}^{(k-1)} \) for \( \vec{w} \)

compute \( \vec{v}^{(k)} = \vec{w} / \|\vec{w}\|_2 \).

Assume \( A = \begin{bmatrix} d_1 & 0 \\ 0 & d_2 \end{bmatrix} \) with \( d_1 > d_2 \), and \( \vec{v}^{(k-1)} = [v_1^{(k-1)}, v_2^{(k-1)}]^T \) with \( v_1^{(k-1)} \neq 0 \) and \( v_2^{(k-1)} \neq 0 \).

  • (a) Express components of \( \vec{v}^{(k)} = [v_1^{(k)}, v_2^{(k)}]^T \) in terms of \( v_1^{(k-1)} \) and \( v_2^{(k-1)} \).
  • (b) Assume, in addition, that \( v_1^{(k-1)} \approx 1 \) and \( v_2^{(k-1)} \approx 0 \). Show that \( \|\vec{v}^{(k)} - \vec{e}_1\|_2 \approx \|\vec{v}^{(k-1)} - \vec{e}_1\|_2^3 \), where \( \vec{e}_1 = [1, 0]^T \). What is the significance of this result? Hint: \( \sqrt{1 + x} \approx 1 + x/2 \) for \( x \approx 0 \).

Solution (a). For simplicity, we denote \(a = v_1^{(k-1)}\in\mathbb{R}\) and \(b = v_2^{(k-1)}\in\mathbb{R}\). Then, we have \[ v^{(k-1)} = [a, b]^T. \] Thus, we have \[ \begin{align*} \lambda^{(k-1)} &= [v^{(k-1)}]^T A v^{(k-1)} = [a, b] \begin{bmatrix} d_1 & 0 \\ 0 & d_2 \end{bmatrix} \begin{bmatrix} a \\ b \end{bmatrix} = [a, b] \begin{bmatrix} d_1 a \\ d_2 b \end{bmatrix} = d_1 a^2 + d_2 b^2, \\ A- \lambda^{(k-1)}I &= \begin{bmatrix} d_1 & 0 \\ 0 & d_2 \end{bmatrix} - \begin{bmatrix} d_1 a^2 + d_2 b^2 & 0 \\ 0 & d_1 a^2 + d_2 b^2 \end{bmatrix} = \begin{bmatrix} d_1 - d_1 a^2 - d_2 b^2 & 0 \\ 0 & d_2 - d_1 a^2 - d_2 b^2 \end{bmatrix}. \\ \end{align*} \] Since, we have \(v^{(k-1)} = w/\|w\|_2\), we have the norm of \(v^{(k-1)}\) is 1. Therefore, we have \(a^2 + b^2 = 1\). Then, we have \[ A- \lambda^{(k-1)}I = \begin{bmatrix} d_1(1 - a^2) - d_2b^2 & 0 \\ 0 & d_2(1 - b^2) -d_1a^2 \end{bmatrix} = \begin{bmatrix} d_1b^2 - d_2b^2 & 0 \\ 0 & d_2a^2 -d_1a^2 \end{bmatrix} = \begin{bmatrix} (d_1 - d_2)b^2 & 0 \\ 0 & (d_2 - d_1)a^2 \end{bmatrix}. \] Since, we know that \(d_1>d_2\) and neither \(a\) nor \(b\) is zero, we have \((d_1 - d_2)b^2\neq 0\) and \((d_2 - d_1)a^2\neq 0\). Thus, we can know that \(A - \lambda^{(k-1)}I\) is non-singular. Therefore, we have the inverse of \(A - \lambda^{(k-1)}I\) as \[ (A - \lambda^{(k-1)}I)^{-1} = \begin{bmatrix} \frac{1}{(d_1 - d_2)b^2} & 0 \\ 0 & \frac{1}{(d_2 - d_1)a^2} \end{bmatrix}. \] Then, we have \[ w = (A - \lambda^{(k-1)}I)^{-1}v^{(k-1)} = \begin{bmatrix} \frac{1}{(d_1 - d_2)b^2} & 0 \\ 0 & \frac{1}{(d_2 - d_1)a^2} \end{bmatrix} \begin{bmatrix} a \\ b \end{bmatrix} = \begin{bmatrix} \frac{a}{(d_1 - d_2)b^2} \\ \frac{b}{(d_2 - d_1)a^2} \end{bmatrix}. \] Hence, we have \[ \|w\|_2 = \sqrt{\left(\frac{a}{(d_1 - d_2)b^2}\right)^2 + \left(\frac{b}{(d_2 - d_1)a^2}\right)^2} = \sqrt{\frac{a^2}{(d_1 - d_2)^2b^4} + \frac{b^2}{(d_2 - d_1)^2a^4}} = \sqrt{\frac{a^6}{(d_1 - d_2)^2a^4b^4} + \frac{b^6}{(d_2 - d_1)^2a^4b^4}}. \] Given that \(b_1>b_2\), \[ \begin{align} \|w\|_2 &= \sqrt{\frac{a^6 + b^6}{(d_1 - d_2)^2a^4b^4}} = \dfrac{\sqrt{a^6 + b^6}}{(d_1 - d_2)a^2b^2}, \\ \vec{v}^{(k)} &= \dfrac{\vec{w}}{\|\vec{w}\|_2} = \dfrac{\begin{bmatrix} \frac{a}{(d_1 - d_2)b^2} \\ \frac{b}{(d_2 - d_1)a^2} \end{bmatrix}}{\dfrac{\sqrt{a^6 + b^6}}{(d_1 - d_2)a^2b^2}} = \begin{bmatrix} \dfrac{a^3}{\sqrt{a^6 + b^6}} \\ -\dfrac{b^3}{\sqrt{a^6 + b^6}} \end{bmatrix}. \end{align} \] Thus, we have the components of \( \vec{v}^{(k)} \) in terms of \( v_1^{(k-1)} \) and \( v_2^{(k-1)} \) as \[ \begin{align} v_1^{(k)} &= \dfrac{(v_1^{(k)})^3}{\sqrt{(v_1^{(k)})^6 + (v_2^{(k)})^6}}, \\ v_2^{(k)} &= -\dfrac{(v_2^{(k)})^3}{\sqrt{(v_1^{(k)})^6 + (v_2^{(k)})^6}}. \end{align} \]

Proof (b).Let \(\varepsilon\approx 0\). Then, we have \(\sqrt{1 - \varepsilon^2} \approx 1 - \varepsilon^2/2 \approx 1\) since \(\varepsilon^2 \approx 0\). Hence, we set \(v^{(k-1)} = \begin{bmatrix} \sqrt{1 - \varepsilon^2} \\ \varepsilon \end{bmatrix} \approx \begin{bmatrix} 1 \\ 0 \end{bmatrix}\). Then, we have \[ \begin{align} \|v^{(k-1)} - e_1\|_2 &= \left\|\begin{bmatrix} \sqrt{1 - \varepsilon^2} - 1 \\ \varepsilon \end{bmatrix}\right\|_2 \\ &= \sqrt{(\sqrt{1 - \varepsilon^2}-1)^2 + \varepsilon^2} \\ &= \sqrt{1 - \varepsilon^2 - 2\sqrt{1 - \varepsilon^2} + 1 + \varepsilon^2} \\ &= \sqrt{2 - 2\sqrt{1 - \varepsilon^2}} \\ &\approx \sqrt{2 - 2(1 - \varepsilon^2/2)} \\ &\approx \sqrt{\varepsilon^2} \\ &\approx |\varepsilon| \\ &\approx 0. \end{align} \] Based on the previous part, we can have \[ \begin{align} v^{(k)} &= \begin{bmatrix} \dfrac{\sqrt{1 - \varepsilon^2}^3}{\sqrt{1 - \varepsilon^2}^6 + \varepsilon^6} \\ \dfrac{- \varepsilon^3}{\sqrt{1 - \varepsilon^2}^6 + \varepsilon^6} \end{bmatrix} = \begin{bmatrix} \dfrac{\sqrt{1 - \varepsilon^2}^3}{(1 - \varepsilon^2)^3 + \varepsilon^6} \\ \dfrac{- \varepsilon^3}{(1 - \varepsilon^2)^3 + \varepsilon^6} \end{bmatrix}= \begin{bmatrix} \dfrac{\sqrt{1 - \varepsilon^2}^3}{1 - 3\varepsilon^2 + 3\varepsilon^4 - \varepsilon^6 + \varepsilon^6} \\ \dfrac{- \varepsilon^3}{1 - 3\varepsilon^2 + 3\varepsilon^4 - \varepsilon^6+ \varepsilon^6} \end{bmatrix}= \begin{bmatrix} \dfrac{\sqrt{1 - \varepsilon^2}^3}{1 - 3\varepsilon^2 + 3\varepsilon^4} \\ \dfrac{- \varepsilon^3}{1 - 3\varepsilon^2 + 3\varepsilon^4 } \end{bmatrix},\\ v^{(k)} - e_1 &= \begin{bmatrix} \dfrac{\sqrt{1 - \varepsilon^2}^3}{1 - 3\varepsilon^2 + 3\varepsilon^4}-1 \\ \dfrac{- \varepsilon^3}{1 - 3\varepsilon^2 + 3\varepsilon^4 } \end{bmatrix},\\ \end{align} \]

6. (a) State the Gerschgorin circle theorem. (HINT: Please state both parts of the theorem.)

(b) Suppose that \( D \in \mathbb{R}^{n \times n} \) is diagonal and \( E \in \mathbb{R}^{n \times n} \) is any matrix. Use the Gerschgorin circle theorem to show that if \[ \|E\|_\infty < \min_{i \neq j} \left| \frac{d_{ii} - d_{jj}}{2} \right|, \] then there is an ordering of the eigenvalues of \( D + E \), \( \mu_1, \mu_2, \mu_3, \ldots, \mu_n \) such that \[ |d_{ii} - \mu_i| < \|E\|_\infty \] for all \( i = 1, 2, 3, \ldots, n \). (HINT: First show that the \( i \)-th Gerschgorin disk is contained in the disk with center \( d_{ii} \) and radius \( \sum_{j=1}^{n} |e_{ij}| \).)

Solution (a).Theorem [Row version Gerschgorin’s Theorem] Let \( A \in \mathbb{C}^{m \times m} \). Consider \( m \) disks in the complex plane: \[ D_i = \left\{ z \mid |z - a_{ii}| \leq \sum_{j \neq i} |a_{ij}| = |a_{i1}| + \ldots + |a_{i,i-1}| + |a_{i,i+1}| + \ldots + |a_{im}| \right\}. \] Then we have the following results.

Proof (b).Firstly, we can know that \[ \|E\|_{\infty} = \max_{k\in\{1, 2, \dots, n\}}\sum_{i = 1}^n |e_{ki}|. \] According to the \(\textbf{Gerschgorin circle theorem}\), if all disks are disjoint, we can know there exists an eigenvalue \(\mu_i\) of \(D + E\) such that \[ \begin{align} |\mu_i - (d_{ii} + e_{ii})| &\leq |e_{i,1}| + \dots + |e_{i, i-1}| + |e_{i, i+1}| + \dots + |e_{i, n}| \\ |\mu_i - d_{ii}| - |e_{ii}|\leq |\mu_i - d_{ii} - e_{ii}| &\leq |e_{i,1}| + \dots + |e_{i, i-1}| + |e_{i, i+1}| + \dots + |e_{i, n}| \\ |\mu_i - d_{ii}| &\leq |e_{i,1}| + \dots + |e_{i, i-1}| + |e_{i, i}| + |e_{i, i+1}| + \dots + |e_{i, n}| \leq \|E\|_\infty. \end{align} \] Now, we want to show that each disk is disjoint. We pick two \( i \) and \( j \) such that \( i \neq j \). Suppose that disk \(D_i\) is centered at \((d_{ii} + e_{ii}, 0)\) and has radius \(r_i = \sum\limits_{k\neq i} |e_{ik}|\). Disk \(D_j\) is centered at \((d_{jj} + e_{jj}, 0)\) and has radius \(r_j = \sum\limits_{k\neq j} |e_{jk}|\). Then, we have \[ \begin{align} 2\|E\|_\infty - |e_{ii}| - |e_{jj}| \leq 2\|E\|_\infty - |e_{ii} - e_{jj}| &< |d_{ii} - d_{jj}| - |e_{ii} - e_{jj}| \leq |d_{ii} + e_{ii} - (d_{jj} + e_{jj})| \\ r_i + r_j + |e_{ii}| + |e_{jj}| &\leq 2\|E\|_\infty, \\ r_i + r_j &\leq 2\|E\|_\infty - |e_{ii}| - |e_{jj}|. \end{align} \] Thus, we have \[ r_i + r_j\lt |d_{ii} + e_{ii} - (d_{jj} + e_{jj})|, \] and we can know that the disks are disjoint. Therefore, there exists an ordering of the eigenvalues of \( D + E \), \( \mu_1, \mu_2, \mu_3, \ldots, \mu_n \) such that \[ |d_{ii} - \mu_i| < \|E\|_\infty \] for all \( i = 1, 2, 3, \ldots, n \) by the second part of the \(\textbf{Gerschgorin circle theorem}\). \( \blacksquare \)